I dont know the code for exponential

  • Thread starter Thread starter frozen7
  • Start date Start date
  • Tags Tags
    Code Exponential
Click For Summary
The discussion focuses on the integration of the function ∫ e^(x^2) x^2 dx, where the user expresses confusion about their approach. The main issue identified is the incorrect substitution of dx into d(x^2) without accounting for the necessary variable, leading to an improper transformation of the integral. It is suggested that the user should integrate by parts instead of substituting variables. The conversation emphasizes the importance of maintaining the correct differential in integration processes. Clarification on the integration method is provided to guide the user toward the correct solution.
frozen7
Messages
163
Reaction score
0
\int \ell^{x^2} x^2 dx

= 1/2 \int \ell^{x^2} x^2 d(x^2)
= 1/2 \int \ell^{t} . t dt (Let x^2 = t )
= 1/2 [t.\ell^{t} - \int \ell^{t} dt]
= 1/2 [t.\ell^{t} - \ell^{t} + c ]
= 1/2 [x^2 \ell^{x^2} - \ell^{x^2} + c]

\ell is actually exponential. I don't know the code for exponential. :)
What`s the problem with the above integration I have done?
 
Last edited:
Physics news on Phys.org
frozen7 said:
\int \ell^{x^2} x^2 dx
= 1/2 \int \ell^{x^2} x^2 d(x^2)
= 1/2 \int \ell^{t} . t dt (Let x^2 = t )
= 1/2 [t.\ell^{t} - \int \ell^{t} dt]
= 1/2 [t.\ell^{t} - \ell^{t} + c ]
= 1/2 [x^2 \ell^{x^2} - \ell^{x^2} + c]
\ell is actually exponential. I don't know the code for exponential. :)
What`s the problem with the above integration I have done?

Well you set t= x^2 but you don't have d(x^2) in the original integral, just integrate by parts you don't really have to make a substitution.
 
frozen7 said:
\int \ell^{x^2} x^2 dx
= 1/2 \int \ell^{x^2} x^2 d(x^2)
= 1/2 \int \ell^{t} . t dt (Let x^2 = t )
= 1/2 [t.\ell^{t} - \int \ell^{t} dt]
= 1/2 [t.\ell^{t} - \ell^{t} + c ]
= 1/2 [x^2 \ell^{x^2} - \ell^{x^2} + c]
\ell is actually exponential. I don't know the code for exponential. :)
What`s the problem with the above integration I have done?
The error lies in line #2, you are changing dx into d(x2), that requires an x, that'll left you with:
\frac{1}{2} \int x e ^ {x ^ 2} d(x ^ 2) not \frac{1}{2} \int x ^ 2 e ^ {x ^ 2} d(x ^ 2). Or you can do a little bit more slowly by letting t = x2. Just try it, and see if you can get to:
\frac{1}{2} \int x e ^ {x ^ 2} d(x ^ 2).
From here, you can do the same and end up with:
\frac{1}{2} \int x e ^ {x ^ 2} d(x ^ 2) = \frac{1}{2} \int x d(e ^ {x ^ 2}).
From here, do you know what to do next?
 
Question: A clock's minute hand has length 4 and its hour hand has length 3. What is the distance between the tips at the moment when it is increasing most rapidly?(Putnam Exam Question) Answer: Making assumption that both the hands moves at constant angular velocities, the answer is ## \sqrt{7} .## But don't you think this assumption is somewhat doubtful and wrong?

Similar threads

  • · Replies 7 ·
Replies
7
Views
2K
  • · Replies 4 ·
Replies
4
Views
1K
  • · Replies 3 ·
Replies
3
Views
1K
  • · Replies 105 ·
4
Replies
105
Views
6K
  • · Replies 5 ·
Replies
5
Views
2K
  • · Replies 2 ·
Replies
2
Views
1K
  • · Replies 36 ·
2
Replies
36
Views
4K
  • · Replies 9 ·
Replies
9
Views
2K
  • · Replies 1 ·
Replies
1
Views
2K
  • · Replies 6 ·
Replies
6
Views
2K